Những câu hỏi liên quan
Dương Thanh Ngân
Xem chi tiết
Đặng Đài Trang
Xem chi tiết
Hinh Sói
Xem chi tiết
Ngọc Anh Phạm
Xem chi tiết
Minh Hiếu
17 tháng 10 2021 lúc 15:35

 sau 20 s vật quay được 10 vòng

⇒ 1s vật quay được 0,5 vòng

⇒ f = 0,5 vòng/s

ta có \(T=\dfrac{1}{f}=\dfrac{1}{0,5}=2s\)

b, đổi 20cm = 0,2 m

\(T=\dfrac{2\text{π}}{\text{ω}}\)⇒ω\(=\dfrac{2\text{π}}{T}\)\(=\dfrac{2\text{π}}{2}\)\(=\text{π}\) rad/s

\(v=r\text{ω}\)\(=0,2\text{π}\)

c, \(a_{ht}=\dfrac{v^2}{r}=\dfrac{0,4\text{π}^2}{0,2}=0,2\text{π}^2\)

Bình luận (0)
Van Doan Dao
Xem chi tiết
Nguyễn Ngân Hòa
26 tháng 12 2020 lúc 17:34

a, Chu kì của chuyển động tròn là: T=\(\dfrac{t}{n}=\dfrac{2}{2}=1\left(s\right)\)

b, Tốc độ dài của chất điểm là: \(v=R.\omega=\dfrac{d}{2}.\dfrac{2\pi}{T}=\dfrac{6}{2}.\dfrac{2\pi}{1}=6\pi\left(\dfrac{cm}{s}\right)\) 

c, Gia tốc hướng tâm là: \(a_{ht}=\dfrac{v^2}{R}=\dfrac{\left(6\pi\right)^2}{R}=\dfrac{36.\pi^2}{3}=12\pi^2\left(\dfrac{cm}{s^2}\right)\)

Bạn tham khảo nha! Không hiểu thì hỏi mình nha.

Bình luận (0)
Sâu Bé
Xem chi tiết
ok ok
Xem chi tiết
minh trần
Xem chi tiết
Nguyễn Ngọc Lộc
18 tháng 9 2021 lúc 11:44

a, Ta có : \(T=\dfrac{1}{f}=\dfrac{1}{5}=0,2\left(s\right)\)

b, Ta có : \(C=2\pi r=0,3\pi\left(m\right)\)

\(\Rightarrow v=\dfrac{5C}{1}=\dfrac{5.0,3\pi}{1}=1,5\pi\left(m/s\right)\)

c,Ta có : \(\omega=\dfrac{2\pi}{T}=10\pi\left(rad/s\right)\)

Bình luận (0)
02-Ngọc Bích
Xem chi tiết
trương khoa
12 tháng 10 2021 lúc 22:32

Đổi 30 cm =0,3 m; 1 phút =60s

a,\(T=\dfrac{1}{f}=\dfrac{1}{\dfrac{N}{t}}=\dfrac{1}{\dfrac{60}{60}}=1\left(s\right)\)

\(\omega=\dfrac{2\pi}{T}=2\pi\left(\dfrac{rad}{s}\right)\)

b,\(v=\dfrac{2\pi}{T}\cdot r=0,6\pi\left(\dfrac{m}{s}\right)\)

\(a_{ht}=\dfrac{v^2}{r}=\dfrac{\left(0,6\pi\right)^2}{0,3}=1,2\pi^2\left(\dfrac{m}{s^2}\right)\)

c, Đổi 10 cm =0,1m

Khoảng cách từ tâm đến điểm được xét =0,3-0,1=0,2(m)

<Rùi tính tiếp>

\(v'=\dfrac{2\pi}{T}r'=0,4\pi\left(\dfrac{m}{s}\right)\)

\(a_{ht}'=\dfrac{v'^2}{r'}=\dfrac{\left(0,4\pi\right)^2}{0,3}=\dfrac{8}{15}\pi^2\left(\dfrac{m}{s^2}\right)\)

 

Bình luận (1)